Dadgarnia

New Member
ارسال ها
1,350
لایک ها
1,127
امتیاز
0
پاسخ : ماراتن جبر (سطح پیشرفته)

اینجا اثبات شد که :-؟ چیز دیگه میخوای؟

کلا سوال بعد! :4:
حداکثر تعداد
ها در عبارت
را بیابید تا
ریشه نداشته باشد.
آهان راست میگی.
https://www.artofproblemsolving.com/community/c6h360818p1974760 فقط عدد هاشون با هم فرق دارن ایده اثبات یکیه.
سوال بعدی رو هم خودت بذار :4:
 

Sharifi_M

New Member
ارسال ها
561
لایک ها
348
امتیاز
0
پاسخ : ماراتن جبر (سطح پیشرفته)

همه ی تابع های
بیابید به طوری که:
اگه اینو دیدی خودت سوال بزار
 
آخرین ویرایش توسط مدیر

Dadgarnia

New Member
ارسال ها
1,350
لایک ها
1,127
امتیاز
0
پاسخ : ماراتن جبر (سطح پیشرفته)

فرض کنید رابطه سوال
باشه. اگه تابع ثابت باشه بدست میاد
یا
. پس فرض می کنیم تابع ثابت نیست. قرار میدیم
. داریم:




رابطه ی بالا رو
می نامیم. داریم:
(۱)


تو رابطه بالا اگه به جای
بذاریم
یه تساوی بر حسب
بدست میاد که چون
ثابت نیست می تونیم نتیجه بگیریم که
یکی از دو مقدار ۱ و ۳ رو داره و بنابر رابطه (۱)
نمی تونه ۳ باشه پس
حالا روابط زیر رو داریم:






همه ی توابعی که توی دو شرط
و
صدق می کنند یه سوال معروفه که فقط جواب های
و
رو داره پس همه ی جواب های سوال بدست اومدن.

---- دو نوشته به هم متصل شده است ----

سوال بعد:
اعداد حقیقی
به گونه ای اند که
(
جایگشتی از اعداد
هستند). ثابت کنید
.
 

AHZolfaghari

Well-Known Member
ارسال ها
935
لایک ها
1,654
امتیاز
93
پاسخ : ماراتن جبر (سطح پیشرفته)

فرض کنید رابطه سوال
باشه. اگه تابع ثابت باشه بدست میاد
یا
. پس فرض می کنیم تابع ثابت نیست. قرار میدیم
. داریم:




رابطه ی بالا رو
می نامیم. داریم:
(۱)


تو رابطه بالا اگه به جای
بذاریم
یه تساوی بر حسب
بدست میاد که چون
ثابت نیست می تونیم نتیجه بگیریم که
یکی از دو مقدار ۱ و ۳ رو داره و بنابر رابطه (۱)
نمی تونه ۳ باشه پس
حالا روابط زیر رو داریم:






همه ی توابعی که توی دو شرط
و
صدق می کنند یه سوال معروفه که فقط جواب های
و
رو داره پس همه ی جواب های سوال بدست اومدن.

---- دو نوشته به هم متصل شده است ----

سوال بعد:
اعداد حقیقی
به گونه ای اند که
(
جایگشتی از اعداد
هستند). ثابت کنید
.

سلام به همه بچه های خوب آیریسک . بعد از مدت ها !!





...

حالا همه رو با هم جمع می زنیم و توجه می کنیم


پس



و



و این یعنی :



و این هم معنی حکم مسئله است .​
 

Dadgarnia

New Member
ارسال ها
1,350
لایک ها
1,127
امتیاز
0
پاسخ : ماراتن جبر (سطح پیشرفته)

سوال بعد:
همه ی سه تایی های حقیقی
را بیابید که
 

Amitis :D

New Member
ارسال ها
21
لایک ها
15
امتیاز
3
پاسخ : ماراتن جبر (سطح پیشرفته)

سوال بعد:
همه ی سه تایی های حقیقی
را بیابید که


حالا توجه کنید که اگر
یک جواب از مساله باشد،
یک جواب مساله خواهد بود. پس بدون کاستن از کلیت مساله می توان فرض کرد که حداقل دو تا از
نامنفی هستند. اگر هر سه نامنفی باشند، تناقضی آشکار است. فرض کنید که مثلا
. در این صورت داریم که
که با نابرابری
در تناقض است.
پس یکی از
منفی است. فرض کنید که داریم:

اکنون:

بنابراین


و

and any permutation is also a solution.
خرکاری بود. P:
 
ارسال ها
10
لایک ها
3
امتیاز
0
پاسخ : ماراتن جبر (سطح پیشرفته)

سوال بعد رو خودتون بزارین.
 
ارسال ها
10
لایک ها
3
امتیاز
0
پاسخ : ماراتن جبر (سطح پیشرفته)

سوال بعد: همه توابع
را بیابید که:

به ازای هر x,y عضو R.
 

Dadgarnia

New Member
ارسال ها
1,350
لایک ها
1,127
امتیاز
0
پاسخ : ماراتن جبر (سطح پیشرفته)

فرض کنید عبارت سوال
باشه. یه عدد دلخواه و مثبت مثل
رو در نظر می گیریم. واضحه که اگه ثابت کنیم
اون وقت برای هر
داریم
. حالا با فرض
داریم:


همینجوری و با استقرا می تونیم ثابت کنیم
حالا اگه توی این رابطه قرار بدیم
بدست میاد:
حالا چون
(lim (sqrt(1/n)+1)^n - Wolfram|Alpha :4:) اگه
رو یه عدد ثابت و مثبت در نظر بگیریم وقتی
رو زیاد کنیم سمت راست از سمت چپ بیشتر میشه که تناقضه پس
که این هم نتیجه میده
.

---- دو نوشته به هم متصل شده است ----

سوال بعد:
فرض کنید
اعدادی حقیقی باشند به طوریکه اعداد
و
و
گویا باشند و
. ثابت کنید
اعدادی گویا هستند.
 

Amitis :D

New Member
ارسال ها
21
لایک ها
15
امتیاز
3
پاسخ : ماراتن جبر (سطح پیشرفته)

سوال بعد:
فرض کنید
اعدادی حقیقی باشند به طوریکه اعداد
و
و
گویا باشند و
. ثابت کنید
اعدادی گویا هستند.
تعریف می کنیم:

بنابراین
ریشه های چندجمله ای
هستند. این چندجمله ای یک چندجمله ای با ضرایب گویاست، پس
اعداد جبری هستند.
از طرفی می دونیم که:
که در اینجا
عددی گویا و مخالف 1-و0 است.
حالا فرض کنید که
عددی گویا باشد. در این صورت:

یعنی:

این نتیجه میده که
ریشه ی یک چند جمله ای درجه ی دوم با ضرایب گویاست. به همین ترتیب میشه ثابت کرد که
ریشه ی یک چندجمله ای درجه ی دوم با ضرایب گویاست.
حالا فرض کنید که
گویا نباشد. (اگر گویا باشد که مساله حل شده است.) در این صورت چون
مضرب گویایی از
هستش، پس
هم گویا نیست. بنابراین،
باید مزدوج جبری هم باشند، و این نتیجه می دهد که
گویاست. حالا داریم:

که تناقض است. پس هر سه باید گویا باشند.
 

Dadgarnia

New Member
ارسال ها
1,350
لایک ها
1,127
امتیاز
0
پاسخ : ماراتن جبر (سطح پیشرفته)

تعریف می کنیم:

بنابراین
ریشه های چندجمله ای
هستند. این چندجمله ای یک چندجمله ای با ضرایب گویاست، پس
اعداد جبری هستند.
از طرفی می دونیم که:
که در اینجا
عددی گویا و مخالف 1-و0 است.
حالا فرض کنید که
عددی گویا باشد. در این صورت:

یعنی:

این نتیجه میده که
ریشه ی یک چند جمله ای درجه ی دوم با ضرایب گویاست. به همین ترتیب میشه ثابت کرد که
ریشه ی یک چندجمله ای درجه ی دوم با ضرایب گویاست.
حالا فرض کنید که
گویا نباشد. (اگر گویا باشد که مساله حل شده است.) در این صورت چون
مضرب گویایی از
هستش، پس
هم گویا نیست. بنابراین،
باید مزدوج جبری هم باشند، و این نتیجه می دهد که
گویاست. حالا داریم:

که تناقض است. پس هر سه باید گویا باشند.
درسته البته اون تهشو میشد با چند جمله ای مینیمال هم گفت که اگه
گویا نباشن
چند جمله ای مینیمالشونه و بقیه اش راحت در میاد.
سوال بعد:
عددی حقیقی است و
اعدادی طبیعی هستند که
اعدادی گویا هستند. ثابت کنید
.
 
بالا